LSAT and Law School Admissions Forum

Get expert LSAT preparation and law school admissions advice from PowerScore Test Preparation.

 Administrator
PowerScore Staff
  • PowerScore Staff
  • Posts: 8917
  • Joined: Feb 02, 2011
|
#47199
Please post your questions below!
 dandelionsroar
  • Posts: 27
  • Joined: Oct 18, 2018
|
#60490
Hi is the reason answer c is wrong because it says "you" instead of keeping it general? I believe it has the same reasoning as the correct answer choice D and the stimulus, which is basically just the contrapositive??
 Adam Tyson
PowerScore Staff
  • PowerScore Staff
  • Posts: 5153
  • Joined: Apr 14, 2011
|
#60763
That's not much of a problem here, dandelionsroar, but there is a big one, and that is that the conclusion in answer C is conditional. We needed a straight-forward contrapositive - the necessary condition will not occur, so the sufficient will not occur. In answer C, the author told us IF the necessary does not occur, THEN then sufficient should not, and that's not the same.

The "you" is meant in general terms, like the third person "one" might be used. It can be interpreted as "if it's not possible", which is what we needed. The conditional nature of the second half of that answer is what kills it for us.

Hope that helps!
 poptart
  • Posts: 1
  • Joined: Feb 17, 2019
|
#62732
Will someone do a more in-depth review of this question?
 Jay Donnell
PowerScore Staff
  • PowerScore Staff
  • Posts: 144
  • Joined: Jan 09, 2019
|
#62742
Hi poptart!

Man, your user name makes me miss the quick breakfasts of my 1990s childhood!

I will try and outline out the conditional reasoning in the stimulus and the correct answer to provide you some clarity, and let me know which incorrect choice(s) you'd like mapped out in addition and I can then do those as well!

As a conditional Parallel question, the foundation of the answer lies in matching the conditional structure to the stimulus.

The stimulus is diagrammed as follows:


P: If make characters more realistic --> viewership will shrink

P: They will maximize their audience (aka not shrink their viewership)
_____________________________
C: They will not make the characters more realistic

This is a fairly straightforward diagram utilizing a contrapositive, and the abstract formula can be condensed to:

A --> B
~B
_____
~A



The correct response in D matches exactly, and can be diagrammed as follows:

P: If the executives were responsible for losses --> losses greater than competitors
P: Losses were less than their competitors (aka not greater than)
___________________________________________________
C: The executives were not responsible for the losses



I hope this helps to bring some clarity!
User avatar
 April30Gang
  • Posts: 12
  • Joined: Feb 24, 2022
|
#94578
This is one of the question I know I'll be skipping on test day. I thought I was a master at parallel reasoning until this question gave me a beat down.

The structure I got was that an action is know to produce a certain effect, but the actors will orchestrate the opposite action because they was the opposite effect.

What are these choices?
 Adam Tyson
PowerScore Staff
  • PowerScore Staff
  • Posts: 5153
  • Joined: Apr 14, 2011
|
#94581
Take another look at it, April30Gang, and you may notice that the argument is conditional rather than causal. Once you see that, and recognize that the conclusion is a contrapositive, it should be a lot clearer which answer you should select. Don't skip a Parallel Reasoning question with conditional reasoning! They are too easy to map out and match the diagram, and that's giving away an easy point!

(Disclaimer: Nothing is really easy on the LSAT. This is just a relative claim - easy compared to a lot of other things you will encounter on the test.)

Get the most out of your LSAT Prep Plus subscription.

Analyze and track your performance with our Testing and Analytics Package.